« first day (41 days earlier)      last day (533 days later) » 
00:00 - 17:0017:00 - 00:00

12:01 AM
0
Q: What is this calculator setting? How do I change it to show decimal always?

Limoenter image description here Everytime I make a calculation for a system of equations it comes out in this format, how do I change the setting so that I always get a decimal?

0
Q: Triangle Area Question

DanQuestion Question - What is the area of the shaded part? I tried doing area ratios but I get to nothing. This is my weak subject. Could someone please help me?

Words such as question are uninformative in titles. Please edit the title so that it better describes the specifics of your question. Do not hesitate to make it longer or include a formula if needed. More tips here. (autocomment)Normal Human 21 secs ago
 
0
Q: What happened to users with helpful questions?

gsamarasQuite often, I have seen questions, helpful ones, that are made by users that do not exist any more? What happened to them? I mean, I do not think they were that bad to be deleted! This was the last question that made me, eventually, ask: How to deploy a war file in Tomcat 7

 
0
Q: How to prove this equation by mathematical induction?

user2982301 + 3 + 3^2 + ... + 3^(n-1) = 1/2(3^n - 1) I am stuck at (3^k - 1)/2 + 3^k and I'm not sure if I am right or not.

0
Q: Prove the closed form of Fibonacci Sequence using Mathematical Induction

Dexterenter image description here Image attached of the closed form of the Fibonacci Sequence.

0
Q: Operators problem

James  WORTHIn $$ L^{2} (\mathbb{R}^2, e^{{-x^2}-y^{2}} dx dy)$$ with subspace $D$ of finite linear combinations of $g_m=(x+iy)^m$ , $m\neq 0$ and integer $(g_0=1)$. Having $$L=\frac{1}{2}\left(\frac{\partial}{\partial x}-\frac{\partial}{\partial y}\right), M=M_{x+iy} $$ Acting on $D$. I cant understand how...

Short title. Title contains problem. Operators problem
0
Q: Fourier transform of a function

Z. AlfataIs that it is a formula that can help to calculate the Fourier transform of $$ f(x) = x^2 e^{-b x^2} e^{-c x \coth(bx)} $$ where $b>0$ and $c\in \mathbb R$. Thanks you in advance

0
Q: Prove that if A is diagonalizable then so is A+kI for any scalars k

Joshua Rocky Lizardii just need help getting started, I'm not sure what tools to use..

0
Q: A triangular piece of land has one side measuring 2ft.

user298236 A triangular piece of land has one side measuring 2ft. The land is to be divided into 2 equal areas by a dividing line parallel to the given side. What is the length of the dividing line? so far this is what i know and it's in one of my quizzes. Can any of you help me answer this please?

0
Q: How to show that $f \in C_0$

The HomeworkerLet $f(x):=e^{x} \int_{0}^{x} e^{-t} g(t) dt$ then I want to show that $$\lim_{x \rightarrow \pm \infty} f(x)=0$$ if $g \in C_0(\mathbb{R}).$ Does anybody know how to show this?

Short title. Short question. How to show that $f \in C_0$
-1
Q: can you measure circumference without pi circles without pi

Anthon ChristensenCan circumference and area of a circle be measured without pi?

 
1:07 AM
0
Q: Norm Product expression

DoeProve the product expression $$\left \| AB \right \|_{U\rightarrow W} \leq \left \| A \right \|_{V\rightarrow W}\left \| B \right \|_{U\rightarrow V}$$ Hint: consider $(AB)u = A(Bu)$ and apply $\left \| Av \right \|_{W}\leq \left \| A \right \|\left \| v \right \|_{V}$ twice

0
Q: Inversion of n x n matrix.

NestiGXHow to make inversion of this (n x n ) matrix? enter image description here

Short title. Short question. Inversion of n x n matrix.
0
Q: Help with a stage in Peter-Weyl proof: that "matrix entry" functions separate points

Latimer LeviosaLet $G$ be a compact Lie group. A "matrix entry function" is one of the form "(i,j)-th entry of the matrix $\rho(g)$" for a Lie group homomorphism $\rho$ from $G$ to a matrix group. (i.e. a representation with a preferred basis) I'd really like it if someone could show me a simple proof (basical...

0
Q: When is a limit of discrete random variables continuous? Is density function analogous to probability fuction? and another question.

WhykaI am trying to answer the following question- Let $X_n\sim Geom(\frac\lambda n)$ and $T_n=\frac{X_n}n$. Show that for $n\rightarrow\infty$ the distribution function of $T_n$ converges to the DF of Z, when $Z\sim Exp(\lambda)$. (1) This brought to the thought- does this mean the limit of $T_...

0
Q: Let $f: A \rightarrow B$ and $g: B \rightarrow C$ be functions. If $f$ onto and $g$ is not onto, then $g \cdot f:A \rightarrow C$ is not onto

JabernetI need help with this proof. I claim it is true, and I want to prove it directly using the definition of onto. Proof: Let $A,B,$ and $C$ be sets, and let f, g be functions s.t. $f:A \rightarrow B$ is onto and $g:B \rightarrow C$ is not onto. Then $\forall b \in B, \exists a \in A$ s.t. $f(a) = ...

 
1:47 AM
0
Q: Is it possible for an improper integral to converge and its series to diverge?

user5853Can an improper Riemann integral converge while its infinite series diverges?

0
Q: Diophantine problem, pythagorean triples, given 3 integers a,b,c

Ameet SharmaI'm trying to prove that there are no 3 positive integers $(a,b,c)$ such that $(a,c), (b,c), (a,a-b+c), (b,a-b+c)$ form 4 pairs of legs of pythagorean triples. Or otherwise to find such a set of integers. ie Prove the system of equations: $$a^2+c^2=w^2$$ $$b^2+c^2=x^2$$ $$a^2+(a-b+c)^2=y^2$$ $...

0
Q: True or False: If A + A^2 is invertible, then A is also invertible

Adam M. A is an n x n matrix here. I understand the proof for A^2 being invertible given that A is invertible, but I fail to see how to incorporate the A + A^2 factor into it. What I have tried so far is a rough factoring to give: A(I_n + A) But that is where I am stuck. Any help is much appreciated!

This site uses MathJax formatting of formulas. More tips here. (autocomment)Normal Human 20 secs ago
0
Q: What does this joke mean?

CarmeisterI saw this written on a blackboard in the math department building the other day: Gas Law: $PV=nRT$ Ideal Gas Law: $(P)(V)=(n)(R)(T)$ I know the ideal gas law is something from chemistry, but I'm assuming this is meant to be some sort of joke involving math. Any ideas?

Tag (notation) should not be the only tag a question has. Please add a tag for a subject area to which the question belongs. (from a bot)Normal Human 21 secs ago
Okay, I'll drop (notation) from that list, @Carmeister
0
Q: A generalization of a theorem about parallel lines in elementary geometry

user74973A theorem in Euclid's Elements states that the line segment between the midpoints of two sides of a triangle is parallel to the third side and is half its length. I have looked in various textbooks in geometry and have not found a generalization of this. If $0 < p < 1$, such as $2/3$ or $\sqrt{...

0
Q: Evaluating integral question

user298244\int _0^1:\frac{\left(5x\right)}{\left(5x^2+6\right)^2}d not sure how to solve this problem

Welcome to Math.SE, user298244. Words such as question are uninformative in titles. Please edit the title so that it better describes the specifics of your question. Do not hesitate to make it longer or include a formula if needed. This site uses MathJax formatting of formulas. More tips here. (from a bot)Normal Human 20 secs ago
0
Q: Norm Product expression

DoeProve the product expression $$\left \| AB \right \|_{U\rightarrow W} \leq \left \| A \right \|_{V\rightarrow W}\left \| B \right \|_{U\rightarrow V}$$ Hint: consider $(AB)u = A(Bu)$ and apply $\left \| Av \right \|_{W}\leq \left \| A \right \|\left \| v \right \|_{V}$ twice Unfortunately, I'm...

 
2:24 AM
0
Q: Why does $A$ similar $D$?

HunLet $A\left( {\begin{array}{*{20}{c}} {{x_1}} & 0 \\ 0 & {{x_2}} \\ \end{array}} \right)\left( {\begin{array}{*{20}{c}} 1 \\ 1 \\ \end{array}} \right) = \lambda \left( {\begin{array}{*{20}{c}} {{x_1}} \\ {{x_2}} \\ \end{array}} \right)$ where $A \in {M_2}(R)$ and all $a_{...

0
Q: Calculating Number of Inputs to operator in RPN

pengowen123I want to know if it is possible to calculate the number of inputs to any operator in Reverse Polish Notation. Suppose I have an operator ^ that takes varying amounts of inputs, specific to each instance of the operator. The input count is not known. All other operators are removed. In this examp...

Welcome to Math.SE, pengowen123. Consider adding a tag for a broader subject area to which the question belongs. Some of these tags might fit. (autocomment)Normal Human 20 secs ago
0
Q: Orthogonal Curve Family Basis

Mathguy0914Given a family of curves $\mathcal{F}_1$ and an orthogonal family of curves $\mathcal{F_2}$, how would one move from these families to an orthogonal curvilinear basis? I have trouble going from the equation representation of the curve families to a vector/matrix representation.

0
Q: Find the largest possible M

Andy YaoLet $m, n, k$ be natural numbers, such that $$m^3=n^2$$ $$m+n=k^2$$ What is the largest possible value for $m$ if $m < 1000$

Short title. Short question. Find the largest possible M
0
Q: A question on minimal polynomials (with relation to some basis)

xlddLet $\mathscr{A}$ be a linear transformation on $n$-dimensional vector space $V$. $0\neq\alpha\in V$. Then it is easy to see that there exists a unique polynomial $m_\alpha(\lambda)$ such that $m_\alpha(\mathscr{A})(\alpha)=0$, and for any polynomial $f(\lambda)$ satisfying $f(\mathscr{A})(\alph...

Questions tend to get more attention when they have a tag for a broad area of mathematics relevant to the question. Some of these tags might fit. (autocomment)Normal Human 21 secs ago
 
2:52 AM
0
Q: How to find the following limits

user289293$\lim_{x\to \infty} \frac{x^2(1+sin^2x)}{(x+sinx)^2}$ I can't figure out how to manipulate this algera so as to get the limit I want. Any hint?

0
Q: Volume generated by a solid revolution

MathematicsFind the volume of solid generated by region in the first quadrant bounded by curve $y=x^2$, below by the x axis and on the right by the line $x=2$ and about the line $x=-3$ How to set up integral in this question? Please help.

Question contains please. Volume generated by a solid revolution
0
Q: Jacobi theta with a matrix

kηivesI would like to evaluate $$ \prod_{i=1}^{N} \sum_{q_i = -\infty}^{\infty} e^{- q_{i} A_{ij} q_{j}} $$ with $A$ a real $N\times N$ symmetric matrix. I know how to compute this when $q$ is continuous (the sum is an integral), and I know how to compute this when $A$ is a scalar (a $1\times 1$, this...

Questions tend to get more attention when they have a tag for a broad area of mathematics relevant to the question. Some of these tags might fit. (from a bot)Normal Human 21 secs ago
 
3:18 AM
0
Q: Find The Volume Enclosed By p=2cos(φ)sin(φ)^2

Bob BoboSo I've been trying to find the volume enclosed by this area for hours and when I do the triple integral with φ from 0 - π, θ from 0 - 2π and p from 0 to 2cos(φ)sin(φ)^2 I get 0!!! QQ Please help me out I'm stumped Here is a graph of the equation

0
Q: Finding possible values for 6 variables in 3 inequations

wjiamI have a problem with solving these 3 symmetric inequations. I want to find positive values for these 6 variables such that the following inequations hold (or show that it is impossible). Please suggest me how to do, or any good tools for this: $\frac{2a}{b+c} \lt \frac{b'+c'}{a'}$ (1) $\frac{2...

0
Q: Direction field and phase portrait question of a system

Big MikeThe system is this: $\frac{dx}{dt}=4x-7y-1$ $\frac{dy}{dt}=3x+6y-12$ Okay i found the equilibrium points to $(0,0)$ and $(2,0)$ but I also read somewhere else that the only way to find the equilibrium points to be trivial is that the det $A$ $\not=0$. This leads me to the phase portrait and di...

Words such as question do not add information to titles. Please edit the title so that it better describes the specifics of your question. Do not hesitate to make it longer or include a formula if needed. More tips here. (from a bot)Normal Human 21 secs ago
0
Q: How can I prove that a continuous line in $\mathbb{R}^2$ is a closed set?

SelfstudyingThis question just occurs to my mind... It is closed clearly from the graph, but I'm wondering if there is a rigorous proof for it.

0
Q: Alternating and absolute convergence

The.SharkConsider the alternating series: sin (x) = x - x^3 / 3! + x^5 / 5! .... If we approximate sin(x) ≈ x, then |x - sin (x) | < | answer | What would be the answer, I tried various methods, but all of them led to wrong answers. Please do explain the working.

This site uses MathJax formatting of formulas. More tips here. (autocomment)Normal Human 21 secs ago
1
Q: Operator Algebras, Operators problem

MarussTIn $$ L^{2} (\mathbb{R}^2, e^{{-x^2}-y^{2}} dx dy)$$ with subspace $D$ of finite linear combinations of $g_m=(x+iy)^m$ , $m\neq 0$ and integer $(g_0=1)$. Having $$L=\frac{1}{2}\left(\frac{\partial}{\partial x}-\frac{\partial}{\partial y}\right), M=M_{x+iy} $$ Acting on $D$. I cant understand how...

Title contains problem. Operator Algebras, Operators problem
0
Q: Show a process is Poisson

Pii Let $Y_i$ be i.i.d. with distribution $P(Y_i = 1) = p$, $P(Y_i = 0) = 1-p$, $0<p<1$. Let $N_t$ be a Poisson Process (with parameter $\lambda$) and let $T_i$ be the arrival of the $i^{th}$ event. Define $$V_t = \sum_{i=1}^{\infty} (1-Y_i) 1_{T_i \leq t} $$ Show that $V_t$ is a Poisson Proc...

0
Q: Question on contraction semigroup and dynamical system

Ever_99I think this is a problem closely related to dynamical system... Let $T(t)$ be a $C^{0}$ semigroup of contractions on a Banach space $X$, and assume that the resolvent $R(\lambda,A)$ of the infinitesimal generator $A$ of the semigroup is a compact operator for some $\lambda>0$. I need to prove ...

Words such as question do not add information to titles. Please edit the title so that it better describes the specifics of your question. Do not hesitate to make it longer or include a formula if needed. More tips here. (from a bot)Normal Human 21 secs ago
0
Q: Compute volume of solid inside both a cylinder and a sphere.

userLet $a$>0 be a fixed positive real number. Consider the solid inside both the cylinder $x^2 + y^2 = ax$ and the sphere $x^2 + y^2 +z^2 = a^2$. Compute its volume. Hint: $$\iint sin^3(x)dx = 1/12cos(3x) - 3/4cos(x) + C$$ Please help I have my exam soon and I don't understand anything about cylind...

0
Q: Tennis Match -Combinatorics problem.

yasirIn a tennis tournament in which every pair has to play a match with every other pair, $10$ players are playing.Find the number of games played. ATTEMPT:-No of ways of selecting $2$ players out of $12$ for forming a pair is same as arranging them in a row and then dividing them into groups of t...

Title contains problem. Tennis Match -Combinatorics problem.
0
Q: How to find all pairs (x,y) of integers such that y^2 = x(x+1)(x+2)?

Rio DuttaHere y^2 is divisible by 12. And satisfying all those conditions y=0 is the only solution. But I can't show it mathematically.

0
Q: What is 1^omega?

DisousaIn Wolfram Mathworld, Ordinal exponentiation $\alpha^\beta$ is defined for limit ordinal $\beta$ as: If $\beta$ is a limit ordinal, then if $\alpha=0$, $\alpha^\beta=0$. If $\alpha\neq 0$ then, $\alpha^\beta$ is the least ordinal greater than any ordinal in the set $\{\alpha^\gamma:\gamma<\b...

Short title. What is 1^omega?
0
Q: Circular Permutations with no restrictions

Nerissa ClarionThere are 4 different Mathematics books and 5 different Science books. In how many ways can the books be arranged on a shelf if there are no restrictions?

0
Q: indefinite differentiation issues

daniel furhangI'm studying for finals and have encountered an issue with a problem. I would greatly appreciate help in finding where I screwed up. $$\int \frac {cos27t^{1/2}dt}{(27t)^{1/2}}$$ I changed it to $$\int (cos27t^{1/2})(27t)^{-1/2})dt$$ and set $$u=27t^{1/2}$$ with $$\frac {du}{dt}=\frac{1}{2}(27t)^{...

0
Q: Find the derivative

LilFind the derivative of tan(radical(1-x)) So I know I have to apply the product rule so wouldn't it be sec^x(rad(1-x))+tanx/2(rad(1-x)) but the final answer says -sec^2(rad(1-x))/2(rad(1-x))

This site uses MathJax formatting of formulas. More tips here. (from a bot)Normal Human 21 secs ago
0
Q: Divergence Theorem evaluate the flux over a sphere

SLMdivergence question Above is the question. I've try to find the divergence of F and parameterize the sphere using spherical coordinates. Below is my work. Then I use online integral calculator(just to avoid human error) to find the result is 100000pi/3, but the result isn't right. Is anything wr...

0
Q: positive define and nonnegative definite function in Bochner&#39;s theorem

JackIn Bochner's thoerem, one condition is $\phi$ is positive definite, which is $\sum_{j=1}^{n}{\sum_{k=1}^{n}{\phi(t_j-t_k)z_j\bar{z_k}}}\geq 0$ for all $z_j\in\mathbb{C}$ and $t_j\in\mathbb{R}$. My question It is the definition of nonnegative definite, not positive definite.

Questions tend to get more attention when they have a tag for a broad area of mathematics relevant to the question. Some of these tags might fit. (autocomment)Normal Human 21 secs ago
0
Q: Divergence Theorem help

The.SharkEvaluate ∬F.ds where F=xi+yj+4zk and S is the surface of the volume: V={(x,y,z)|x^2+y^2≤9,0≤z≤1} Find divF ∬F.ds = ∫∫∫ divF dxdydz = ?? Please explain in detail how to get the answer.

Words such as help are uninformative in titles. Please edit the title so that it better describes the specifics of your question. Do not hesitate to make it longer or include a formula if needed. This site uses MathJax formatting of formulas. More tips here. (from a bot)Normal Human 21 secs ago
 
4:48 AM
0
Q: Question on the distribution of eigenvalues in a square matrix with random entries.

Armen AghajanyanLet square matrix $A$ of size $n \times n$, have entries that have been independently sampled from a uniform distribution between $[a_1,a_2]$. The question I have is what distribution will the eigenvalues fall under? My Approach: My exploration began by looking at the characteristic polynomial ...

0
Q: Find the revenue and demand functions for the marginal revenue

user298265Find the revenue and demand functions for the marginal revenue $\frac{dr}{dx}=50+7x-37x^2$ How can I solve this problem?

0
Q: Prove by induction closed form of Fibonacci Sequence

Dexter43Fibonacci Numbers fn are defined recursively as fn = fn-1 + fn-2 for n > 2 and f1 = f2 = 1. They also admit a simple closed form: √5𝑓𝑛 = (1+√5/2)^2 - (1-√5/2)^2 Prove this formula by using induction.

Welcome to Math.SE, Dexter43. This site uses MathJax formatting of formulas. More tips here. (from a bot)Normal Human 21 secs ago
 
5:14 AM
0
Q: $\hat{K}\subset co(K)$

EpsilonDelta Let $K$ be a compact subset of $\mathbb{C}$. Then the polynomially convex hull $\hat{K}$ and the convex hull $co(K)$ of $K$ are defined as follows: $\hat{K}=\{z\in \mathbb{C}:|p(z)|\leq max_{\zeta \in K} |p(\zeta)|$ for all polynomials $p\}$. $co(K)=\{z\in \mathbb{C}:L(z)\leq max_{\...

A title should not be all-MathJax; having some plain text helps with search and navigation. (from a bot)Normal Human 20 secs ago
0
Q: Find the derivative of this question

LilFind y': yx^3=4e^(xy) how do I go about finding this derivative? Step by step explanation please!

Words such as question are uninformative in titles. Please edit the title so that it better describes the specifics of your question. Do not hesitate to make it longer or include a formula if needed. More tips here. (autocomment)Normal Human 20 secs ago
0
Q: Why is the Sydney Harbour Bridge a Parabola?

CanisFamiliarisDoes its shape have a purpose? If so, what is it?

 
5:56 AM
0
Q: Trouble understanding Product Notation

user270494 I'm trying to determine the quadrature formula when the interval is [−2, 2] and the nodes are −1, 0, and 1. However, I get stuck on this step because I don't understand how the attached image is equal to $1/2x(x-1)$ My attempt is, $(x - 1)/(-1 - 1)$ and $(x - 2)/(-1-2)$ but that is completely...

0
Q: Regarding self complementary trees if such exist.

AnonymousCan anybody please tell me whether there are any self complementary trees and if so, how many and what are the conditions and the properties it holds? Thank You.

Short question. Question contains please. Regarding self complementary trees if such exist.
0
Q: Is there a means of finding an infinite sum by means of altering it into an integral?

Orange Julius CesaerIf you are given a sum, say $$\sum_a^b f(x)$$ with $a,b\lt \intfty$ Is there a means of solving for this sum by means of integration? I am familiar with sophomores dream.

0
Q: What are the equilibrium solutions of this first order linear system?

Big Mike$$\frac{dx}{dt}=4x-7y-1$$ $$\frac{dy}{dt}=3x+6y-12$$ one part of my notes says if the determinant gives you some number other than zero then there is only one equilibrium solution (the origin). However another part of my notes says that the equilibrium solutions are just setting each equatio...

0
Q: An example of a divergent function that is bounded and continuously differentiable.

MC989I want to show that the function $f(x)=(-1)^x$ is such a function. Is it sufficient to show that the limit of two subsequences of $x$ are not equal (i.e. $1$, $-1$)?

0
Q: Cardinality of a languages and Model

MCSHIn the Van Dalen's Logic and Structure how is cardinality of a language defined? what about models?

0
Q: Definition of bar resolution

VincentI am reading Weibel's book about homological algebra. In section 6.5, the bar resolution, he uses some notation I really do not understand. So given G, a group, what is $[g_1\otimes...\otimes g_n]$? And what is $[g_1|...|g_n]$?

 
6:22 AM
0
Q: If $f:\mathbb{R}^{2}\rightarrow\mathbb{R}$ is continuous in each variable separately, then it is continuous.

neelaThere is incorrect statement If $f:\mathbb{R}^{2}\rightarrow\mathbb{R}$ is continuous in each variable separately, then it is continuous. Counterexample is $$\frac{2xy}{x^{2}+y^{2}}$$ as it is discontinuous at $(0,0).$ Now i am searching the condition under which the above stated incorrect st...

 
6:46 AM
0
Q: Adapted class of flat module

VincentI am wondering whether flat module is a adapted class to the functor F(M)=M$\otimes$ N. Currently I am stuck by showing every module is a quotient of flat-module. This is too bizarre for me, I am not sure whether this is true. Could any help me?

 
6:58 AM
0
Q: How do you prove $\lim_{x\to\infty} 1/n^{1/n}$ using only basic limit theorems?

MC989How do you prove $\lim_{x\to\infty} \frac {1}{n^{1/n}}$ using only basic limit theorems? I thought it was $0$, but my book lists the solution as $1$. How come?

0
Q: A question on the decomposition of the group algebra $\mathbb{C}[G]$ of a finite group $G$

jeffreyI am am very confused about a fundamental result in representation theory of finite groups. Please let me first introduce the setting. Let $G$ be a finite group. The group algebra $\mathbb{C}[G]$ is a semisimple ring by Maschke's theorem. Hence, there is an isomorphism $$ \mathbb{C}[G]\cong U_1\...

Title contains question. [A question on the decomposition of the group algebra $\mathbb{C}[G]$ of a finite group $G$](math.stackexchange.com/q/1573241)
 
7:21 AM
0
Q: Real Analysis from Richard Bass Exercise 10.6

user2856673Suppose that fn and f are measurable functions such that for each e> 0 we have ∑μ({x:|fn(x)-f(x)|>e})<1. Prove that fn -> f a.e.

Title contains hard, exercise. Short question. Real Analysis from Richard Bass Exercise 10.6
0
Q: Ladder operator identity

shinobi20Define $n=(x + iy)/\sqrt{2}L$ and $\overline n=(x - iy)/\sqrt{2}L$. Also, $\partial_n$ = $L(\partial_x - i \partial_y)/\sqrt{2}$ and $\partial_\overline n$ = $L(\partial_x + i \partial_y)/\sqrt{2}$. with $\partial_n=\partial/\partial n$, $\partial_x=\partial/\partial x$, $\partial_y=\partial/...

0
Q: If X is an finite set, show that the following conditions on a function $f:X\to$X are equivalent.

bellaI start the proof off but saying we must show that $f:X\to$X must be injective, surjective, and bijective.

 
7:46 AM
0
Q: boundary on R^3 about Stoke's thm

JAEMTOLet S be the union of two surfaces, $S_1$ and $S_2$, where $S_1$ is the set of $(x,y,z)$ with $x^2+y^2 =1$ ,$0<= z <=1$ and $S_2$ is the set of $(x,y,z)$ with $x^2+y^2+(z-1)^2 =1$, $z>=1$. Set $F(x,y,z)$=$(zx+z^2y+x)i + (z^3yx+y) +(z^4x^2)k$ Compute $\int\int_S curlF *dS$ (*: inner product) S...

Questions tend to get more attention when they have a tag for a broad area of mathematics relevant to the question. Some of these tags might fit. (autocomment)Normal Human 1 min ago
 
0
Q: Question limit?

HunterIs it my mistake that people are not able to answer questions that I ask, or they do not bother to answer some petty questions? I sent a request to enable my question limit, according to the admins, the questions need to be helpful to others in the community, which I disagree from, as many questi...

 
8:15 AM
-1
Q: Calculus 2 Help!! (infinite series)

ahmburdanselI need help with these infinite series! Having a hard time figuring out where to start:

Welcome to Math.SE, ahmburdansel. Words such as help are uninformative in titles. Please edit the title so that it better describes the specifics of your question. Do not hesitate to make it longer or include a formula if needed. More tips here. (from a bot)Normal Human 41 secs ago
 
0
Q: Should the "Save Profile" button be always enabled?

Maroun MarounWhen I edit my profile and make some changes, I get the "Your profile has been saved successfully" message. However, the "Save Profile" button is kept enabled and clicking on it reloads the message even if no changes were made: I don't know if it's by design, but I think it's a bit confusing; ...

 
8:36 AM
0
Q: What is $\pi-3$?

Vedant ChandraWhen I run this in WolframAlpha, it directly subtracts $3$ from a decimal approximation of $\pi$, giving $$0.1415926535897932384626433832795028841971693993751058...$$ However, in pure mathematical terms, what is $\pi-3$? Does a closed form exist for it (excluding '$\pi-3$')? Is it irrational, an...

Short title. What is $\pi-3$?
0
Q: Formulation of numerical schemes

MarkPlease I want to know what kind of trigonometric basis I can use instead of polynomial basis for construction of numerical scheme.

Short question. Question contains please. Formulation of numerical schemes
 
9:04 AM
0
Q: Roots of a $x^2-1$ over $\mathbb{z}_7$?

user139985 Is it enough to show that no single element of $\mathbb{Z}_7$ squared is equal to -1? Or is there more to it than that?

 
0
Q: Captcha impossible on iPad

GrantlyIn some cases the captcha asks you to copy some code into a new text box, but there is no way of doing this on an iPad (1). It is also not possible to request a new captcha as the same paradigm applies to the new one offered. Is there a way around this annoying captcha such that I may contribut...

 
0
Q: Clarification about notation

Aloysius GodinhoI am confused with the notation $\mathbb{Q}[x,y]/(x)$. I understand that $\mathbb{Q}[x,y]$ is the ring of polynomials in two variables over $\mathbb{Q}$. So $(x)$ has to be an ideal. Could anyone explain what $(x)$ means? Thank you

 
0
Q: what is this "cite" in the same line as edit and close

anna vI must have missed something. What is this "cite" in the same line as "edit" and "close" in a question, and "edit" and "flag" in an answer. Cite what and where?

 
9:19 AM
0
Q: Maximize the number when travel from A to B

Khanh DinoI have a math from my teacher and I can't find the answer, please help me: - Tom has to deliver oranges from A to B with distance (d - Ex: d=1000) and total oranges (n - Ex: n=3000) and vehicle capacity (m - Ex: m=1000). And vehicle will loses 1 orange every 1 unit of distance, the question is fi...

Welcome to Math.SE, Khanh Dino. Consider adding a tag for a broader subject area to which the question belongs. Some of these tags might fit. (autocomment)Normal Human 21 secs ago
0
Q: A question about the splitting of the polynomial $x^7-2$

Aloysius GodinhoSo my question is suppose $\xi =\exp{\frac{2 \pi i}{7}}$, then does the polynomial $x^7-2$ reduce in the field $\mathbb{Q}[\xi]$? If so how?

Words such as question do not add information to titles. Please edit the title so that it better describes the specifics of your question. Do not hesitate to make it longer or include a formula if needed. More tips here. (from a bot)Normal Human 22 secs ago
0
Q: can anyone help me solve this?

CCKKLLGGLet {W_t,t larger or equal 0} be a standard Brownian motion and X_t=mut+sigmaWt.Where mu and sigma are constants, and sigma greater than0. 1.What is the mean of vector=(X1,X2)? 2.What is the covariance of this vector? 3.What is the joint distribution of this vector? 4.Let a b c be constant.W...

 
0
Q: Cannot add a comment using iPad

GrantlyThe add comment button seems to do nothing, rendering it impossible to comment on a question. iPad 1, latest possible iOS for hardware 5.1.1, using Safari. Perhaps the functionality has changed for commenting in certain scenarios? Why should I assume 100% that this is a bug? Please do not edit m...

 
Words such as anyone, help are uninformative in titles. Please edit the title so that it better describes the specifics of your question. Do not hesitate to make it longer or include a formula if needed. This site uses MathJax formatting of formulas. More tips here. (autocomment)Normal Human 34 secs ago
 
9:45 AM
0
Q: How to calculate the controls of this Bézier curve?

ostal123How to calculate the controls of this curve if I know three points: start, one on the curve and the end? Here is the curve with the coordinates I know: The curve with the points I've never done this kind of calculation before, so please be patient and explain it.

 
10:03 AM
0
Q: On Greatest Common Divisors

YYGLet $F$ be a field and $r$ and $s$ positive integers. Prove that, in $F[x]$, $GCD(x^r-1,x^s-1)=x^{GCD(r,s)}-1$. If $r$ and $s$ were known numbers, I could be able to attempt the problem, but I don't know how to do about this.

 
10:23 AM
0
Q: Finding the Common Ratio given the sum of first two terms and the infinite sum

Joyce FengThe sum of the first two terms of a geometric sequence is 1100, and the sum of the infinite sequence is 3600. Find the common ratio given that r is positive.

0
Q: R is an injective R-module .We will have Ann(I∩J)=Ann(I)+Ann(J) for every two ideal like I and J

maryR is an injective R-module . For every two ideals I and J we will have Ann(I∩J)=Ann(I)+Ann(J). I made an effort for an hour but it was not helpful.

0
Q: smallest, non-zero eigevalue of a sparse hermitian matrix using matlab's eigs

Kosha MisaI'm looking for the first 10 smallest non-zero eigenvalue in a symmetrical, sparse, hermitian Matrix A (3000x3000). There are also a lot of zero-eigenvalues in the eigenalue-spectrum which are not of interest. Matlab's eigs lets me define a sigma (starting point), but the result always gives me...

 
10:53 AM
0
Q: If f''(x) is continuous, then does this mean that f'(x) is continuous and f(x) is continuous? thanks

Niamh45If f''(x) is continuous, then does this mean that f'(x) is continuous and f(x) is continuous

0
Q: How can we calculate the degree of angle made by the matches?

s.a.a I was playing a game on my phone when a question pop up on my screen coming from one of my best mathematics masters: If we know that all of the matches are in the same size, what would be the alpha's degree?

Tag (contest-math) should not be the only tag a question has. Please add a tag for a subject area to which the question belongs. (autocomment)Normal Human 31 secs ago
0
Q: Prove that $\int_0^1 f(x)dx=0$ if $f(\frac{1}{n})=1$ for $n=1,2,3,\ldots$ and $f(x)=0$ for all other $x$

luka5z Prove that $\int_0^1 f(x)dx=0$ if $f(\frac{1}{n})=1$ for $n=1,2,3,\ldots$ and $f(x)=0$ for all other $x$. Lemma: If $g:[a,b]\rightarrow \mathbb{R}$ is a function such that $f(x)=\mathbb{1}_{\{c\}}$ for some $a<c<b$, then $\int_a^b g(x)dx=0$. Proof of the lemma: Consider a partition of ...

0
Q: Product of the radii

Amad27 $A_1$ and $A_2$ are two circles in a plane. The common external tangent to $A_1$ and $A_2$ consists of length $2017$. The common internal tangent consists of length $2009$. Find $r_1*r_2$ the product of the radii. This is fairly complicated. The solution uses $(r_1 - r_2)^2 + 2017^2$, but ...

Short title. Product of the radii
0
Q: Find the period of a state

whoisitThe question is question description And the answer isThe only state with period> 1 is 1, which has period 3. I don't understand why other states like 2,3,5,6 are not with period 3, they can also take 3 steps back to themselves, can't them?

0
Q: Does this prove that two lines are parallel?

user298322I was recently given this problem: Suppose we have a triangle ABC and let there be a cevian AD. Let the foot of the perpendicular from B to AD be P and let the intersection of the extension of BP to AC be Q. Prove that DQ and AB are parallel. So I extended a line from D parallel to AC and let it...

0
Q: prove that $K(G) \geq i(G)$ where $K(G)$ is dominating number of $G$

ghazaleprove that $K(G) \leq i(G)$ where $K(G)$ is dominating number of $G$ and $i(G)$ is indepent dominating number.

 
11:25 AM
1
Q: Should we close a question which may little "basic"? If so, which reason?

Kevin GuanThere's some very basic questions, so should we close them? What I meant is, for example here's a question closed as a problem that can no longer be reproduced or a simple typographical error. However, I think that reason is incorrect in this case. Because I think Can no longer be reproduced an...

 
0
Q: Find all x∈ℤ satisfying each of the following equation

1081I am doing some exercises on groups and I am having trouble trying to show why the answer to: 5x+1 = 13 mod 23 [where the equals sign means congruent] has the answer: 18 + 26Z [where Z is the set of integers] can anyone please explain? Thanks for you help.

0
Q: Graph problems(graphic)

soullessI have been doing some exercise on graphs and I have been working on it for days and I don't even understand the solution and the proofs. For question 44 to 47, can anyone lend me a hand? The text solution that I am having is very confusion and i can't visualize or wrap my head around it.. Thanks...

Short title. Title contains problem. Graph problems(graphic)
0
Q: MathJax editor for printing

LegolasI am trying to write an exam paper for my students on computer but i am trying to find a mathjax editor that i can write them and print them.I tryed to search on internet but i did not find anything,so any suggestion would be appreciated.

 
11:50 AM
-2
Q: Publishing GitHub content as regular web pages

AndyI have posted some stuff to GitHub, including an XHTML file that I would like to be available as a normal web page. This would be in addition to the standard Github access that would still be required for maintaining the codebase. So, the question is, can this be done, and, if so, how?

0
Q: Should I edit out the background in the question when I see the answers doesn't use this information?

OokerShould a dismissal from PhD in graduate application be listed as academic misconduct? I find the background part in this question doesn't give useful information, or at least the answers don't use it. Should I delete the whole part to make the question short and to the point?

0
Q: How can I correct a tag?

sawaI just created a tag called "ruby2.3", but for consistency with other versions, I should have made it "ruby-2.3". I would like to rename the tag rather than creating a synonym. How can I do that?

 
-1
Q: Saturated submodule

AlopisoIm trying to solve this exercise, but I can't... Let S be a multiplicatively closed subset of A.M a A-module,N a submodule of M and $\phi_M:M\longrightarrow S^{-1}M$ homomorphism given by $\phi_M(m)=\frac{m}{1}$. Therefore, the following are equivalent: There exists a $S^{-1}A$-submodule P of...

Short title. Saturated submodule
0
Q: Sum of Sequence

Noam$c_{n} = \frac{1+(-1)^{n}}{2}$ $S_{n} = c_{1} + c_{2} + c_{3} + ... + c_{n}$ Proove that $lim \frac{S_{n}}{n} = \frac{1}{2}$ These are my steps $\rightarrow S_{n} = \frac{n}{2n}(\frac{1+(-1)^{n}}{2}) = \frac{1+(-1)^{n}}{4}$ $\frac{1+(-1^{n})}{2}$ is $2$ or $0$, so the lim of the sum is $\fr...

Short title. Sum of Sequence
0
Q: For which $x$ is $2^{x+1}-2 \equiv 0 \pmod{29} \quad and \quad 2^{x+1}-4 \equiv 0 \pmod{28}$

redelectronsIn $$2^{x+1}-2 \equiv 0 \pmod{29} \quad and \quad 2^{x+1}-4 \equiv 0 \pmod{28}$$ How can I find for which $x$ this holds true. $x$ is a positive integer

 
12:17 PM
0
Q: f is concave, g:R->R is decreasing, prove that g*f (composition) is convex

Eddie YuPlease prove that If function f is concave, g:R->R is decreasing, then g*f (composition) is convex

0
Q: How to solve y''-y=-2cosx, y(0)=1, y'(0)=0

SergeiHow to solve the following equation? y''-y=-2cosx; y(0)=1, y'(0)=0; I have to find a solution to the Cauchy problem, but I can not:(

 
12:29 PM
0
Q: Two equivalent vectors groups

xlddLet $v_1,\cdots,v_n$ and $w_1,\cdots w_n$ to be two equivalent vectors groups in $\mathbb{R}^m$, that is, $v_i$ can be linearly expressed by $w_1,\cdots w_n$, and $w_j$ can be linearly expressed by $v_1,\cdots,v_n$. Can we show that there exists an invertible matrix $P$ such that $$(w_1,\cdots w_...

0
Q: mathematics puzzle

algebra1The Big and the Small Empire are both rectangular islands and divided into rectangular landscape. In each province there is a road that runs along one of the diagonals. On each island exist roads that make a closed route, which does not go through any point several times. The picture shows the Li...

Tag (problem-solving) should not be the only tag a question has. Please add a tag for a subject area to which the question belongs. (autocomment)Normal Human 25 secs ago
0
Q: Probability of placement

thmwFind the probability that the random placement of n balls in m boxes, there is exactly s boxes containing exactly k balls. (assume that, s*k <= n).

Short title. Short question. Probability of placement
0
Q: Do we need parenthesis in lim?

studentIs  $\lim_{x\to 0} x+x^2=(\lim_{x\to 0} x)+x^2=0+x^2=x^2$ or $\lim_{x\to 0} x+x^2=\lim_{x\to 0} (x+x^2)=0+0^2=0$?

0
Q: Chain and anti chain problem

user290335Hi Can anyone help on this? Consider the set which consists of all sets whose elements are natural numbers. I need to define an infinite chain and anti chain, on this set, where the ordering is by inclusion. So I did: Anti chain - the set of prime numbers, since prime numbers are a subset of ...

Short title. Title contains problem. Chain and anti chain problem
0
Q: Examples compact sets

lars111At the moment I try to understand the topic "proving compact sets". 2 examples: I want to ask, if my assumptions/conclusions are right. Example 1: $(x1-1)^3 + x2 \le 0,x2\ge0$ This set closed because there are just greater/less than equal signs. This set is not bounded, because if x1 goes to ...

 
12:55 PM
0
Q: r e n a m e spaces

HennesWe have two tags which seem to be text besed: tabs & spaces However tabs is defined for the graphical user interface feature that allows multiple documents to be contained within a single window and spaces is tagged as the OSX moniker for "Virtual Desktops". There were quite a few questions tag...

 
0
Q: Limit problem with fifth root

Lovro How to solve this limit without L'hopital rule and Taylor series?

Short title. Title contains problem. Short question. Limit problem with fifth root
0
Q: What class of problem is a set of equations using inequalities and if-then-else?

dawCan you please identify what class of problem this is so that I can research algorithms for solving it please? Its a a set of linear equations and inequalities/constraints looking like this: aq = 3*ct bq = 1*dt + 2 cq = 1 dq = 2 at = if (aq >= bq) then 1 else 0 bt = if (aq < bq) then 1 else 0 c...

0
Q: Compute $\lim_{x\to 0} \frac{x-sinx}{x-tanx}$ with L'hopital rule.

Ilan Aizelman WSCompute $\lim_{x\to 0} \frac{x-sinx}{x-tanx}$ with L'hopital rule. Well, I know that I need to use the Quotient rule first $(\frac{f}{g})'$. but then, what do I have to do next?

 
1:13 PM
0
Q: tag [writedata] shouldn't be written

QuillThe writedata tag seems to be used for cases where data is written to a file, loosely used throughout different languages, not pointing to a singular function/method. There's no tag wiki/summary either. Immediately looking at the question, I see r, swift and python questions. There's only 18 qu...

0
Q: Not able to lookup remote EJB From different Web application that is deployed in different machine(or on different server)

AnandRamI created EJB Application and deployed on Jboss6.1 EAP and it's working fine, tried lookup from standalone application in the same jvm and it's also working fine. But not success when i tried from different web application deployed another server(and tried from standalone application of remote ...

 
0
Q: maximum chain and maximum anti chain problem

user290335Can anyone help me on this? Consider the divisibility partial order on A, so a ≤ b if b = ma for some integer m. Is this a total order? I need to write a maximum chain and maximum anti chain if they exist. I Know that to be a a partial order, it need to obey: If a ≤ b and b ≤ a then a = b ...

0
Q: Weirdly defined ball compact in $C^1([0, 1])$

StudentConsider$$B := \left\{u \in C^2([0, 1]) : \sum_{i=0}^2 \sup_{x \in [0, 1]} \left|u^{(i)}(x)\right| \le 1\right\}$$as a subset of $C^1([0, 1])$. How do I see that it is compact in $C^1([0, 1])$?

Title contains weird. [Weirdly defined ball compact in $C^1([0, 1])$](math.stackexchange.com/q/1573483)
0
Q: Is my proof for $\sqrt{2}$ Irrationality correct?

RestlessC0braI use strong induction on $p$. Proof. We want to show that $\forall q\in \mathbb{N} \big[q>0 \rightarrow \neg\exists p\in\mathbb{N}\big(p/q=\sqrt{2}\big)\big]$. Let $q$ be arbitrary natural number and $q>0$. Inductive Hypothesis. Let $k\in\mathbb{N}$ and $k<p$ such that $\big[q>0 \rightarrow \n...

Tag (proof-verification) should not be the only tag a question has. Please add a tag for a subject area to which the question belongs. (autocomment)Normal Human 21 secs ago
 
1:34 PM
0
Q: Find det in terms of k

Adam a) Find det(B) in terms of k; b) For what value(s) of k are the column vectors of B linearly dependent; enter image description here

Short title. Short question. Find det in terms of k
0
Q: Problem with $F$ bounded $\rightarrow$ $F$ continuous

SkillsI have to proof: $F$ bounded in the ball $\rightarrow$ $F$ continuous Suppose (absurde) exists a sequence $\{ x_n \}$ with $\left \| x_n \right \|=1$ $\forall n \in N$ such that $\frac {F(x_n)}{\left \| F(x_n) \right \|} \xrightarrow{n \to \infty} + \infty$. So: $$\frac{x_n}{F(x_n)}= \frac{x_n}...

 
0
Q: More options for low quality posts

Federico PoloniAs I have recently passed the 10k reputation threshold, Stack Exchange is now asking me for more moderation work, including handling low quality posts. The interface for this task seems inadequate to me. When a low-quality post is reported, I have only four options: Looks ok, Edit, Recommend del...

 
0
Q: Prove the triangle is equilateral

viratLet ABC be a triangle. Let Γ be it’s circumcircle, and let I be it’s incenter. Let the internal angle bisectors of ∠A,∠B,∠C meet Γ in A',B',C' respectively. Let B'C' intersect AA' at P, and AC in Q. Let BB' intersect AC in R. Suppose the quadrilateral PIRQ is a kite; that is, IP = IR and QP = QR....

Question contains please. Prove the triangle is equilateral
 
1:49 PM
-2
Q: Add blocker bug in stackoverflow

jonasnasStackoverflow is not showing a page when used with add blocker. Disabling plugin makes page work correctly.

0
Q: Group of $r$ people at least three people have the same birthday?

Mithlesh UpadhyayI've posted a problem Group of $r$ people at least three people have the same birthday? I'm not getting properly. I've tried it and answer should be $$\displaystyle 1- (\frac{365\cdot364 \cdots(365-r+1)}{365^r} +{r\choose 2}\cdot \frac{365\cdot364\cdot363 \cdots (364-(r-2) +1)}{3...

 
2:15 PM
0
Q: Combinations and sums

bjcsbits028In how many ways can be get a sum greater than x for n distinct numbers where each number can be between 1 and (x-1), both inclusive? Foe example- For x = 5 and n = 3, the required combinations are (1,2,3), (1,2,4), (1,3,4) and (2,3,4) i.e. total 4 ways. Similarly for x = 6 and n = 3, the requir...

0
Q: Entropy of a character in a String

lonesomeTaking into account the Shannon entropy, I was wondering that, if we have a String like 1122344444455 , is this possible to find out the entropy of digit 4 in this String? In other words, I would like to know if we can find a way to measure the degree of uncertainty of occurrence of digit 4 in th...

0
Q: limit of recursice sequence

Noam$0 \leq a\leq \frac{1}{4}$ $a_{1} = a,$ $a_{n+1} = a + (a_{n})^{2}$ I have to proove that the sequence has a limit, and to find the limit $a_{n}$. Actually, I have no idea what to do. I know that $a_{2} > a_{1}$ and for $n>2 ,$ $a_{2} > a_{n}$ But what's next ? Thanks.

Questions tend to get more attention when they have a tag for a broad area of mathematics relevant to the question. Some of these tags might fit. (autocomment)Normal Human 33 secs ago
0
Q: question on Polynomials

PARTHOShow that there are infinitely many triples (x,y,z) of integers, such that x^3 + y^4 = z^31.

Words such as question are uninformative in titles. Please edit the title so that it better describes the specifics of your question. Do not hesitate to make it longer or include a formula if needed. More tips here. (autocomment)Normal Human 33 secs ago
0
Q: $\gcd(a,b) * [a.b] = ab$

user3210476Ho do I go about solving such type of problems? $gcd(a,b) * [a.b] = ab$ where $[]$ represents greatest integer function. Just want a direction to start..

A title should not be all-MathJax; having some plain text helps with search and navigation. (autocomment)Normal Human 45 secs ago
0
Q: Need help in Calculus of Variation.

zafranGiven problem is $J[y] =\int_{0}^{x_1}y'^2dx$ with $y(0)=0$ and $y(x_1)=-x_1-1$. After solving Euler Lagrange equation I got $y=Ax+B$ . And using first boundry conditon I got $y=Ax$ We have transversatity condition $[F+(\phi'-y')F_{y'}]=0 $ at $x=x_1$. But solving this I am getting $A=0$ ...

Words such as help are uninformative in titles. Please edit the title so that it better describes the specifics of your question. Do not hesitate to make it longer or include a formula if needed. More tips here. (from a bot)Normal Human 22 secs ago
0
Q: Upper triangular matrix and sum on diagonal

Joe MahanashaMatrix is upper triangular and for each $k = 1, 2, ..., n$ we got: $rank(A - k \cdot I_{n}) < n$. Find the sum of the elements on the main diagonal.

0
Q: Calculate limit

yousef saffaI need to calculate limit $$‎\lim‎_{ ‎r\rightarrow ‎\infty‎}‎‎\frac{\Gamma(r\alpha)}{\Gamma((r+1)\alpha)}‎‎$$ where $0<\alpha <1$ and $\Gamma(.)$ is Gamma function. with thanks in advance.

Short title. Calculate limit
0
Q: Two conditions of quasi-concave function

Eddie Yuf:R^n->R prove that the two statements are equal 1) For all x,y in R^n, and for all t in [0,1], f(tx+(1-t)y)>=min{f(x),f(y)} 2) For all k in R, {x : f(x)>=k} is a convex set.

0
Q: Question on Orthogonal Complement on Hilbert Space

Shrey GuptaLet M and N be linear subspaces of a Hilbert space $H$ with $M \perp N$. Show that $M^{\perp\perp}\perp N^{\perp\perp}$. Is it true that $M^{\perp}\perp N^{\perp}$, or $M^{\perp\perp\perp}\perp N^{\perp\perp\perp}$ ?

Welcome to Math.SE, Shrey Gupta. Words such as question are uninformative in titles. Please edit the title so that it better describes the specifics of your question. Do not hesitate to make it longer or include a formula if needed. More tips here. (from a bot)Normal Human 29 secs ago
 
2:41 PM
0
Q: Does such function exist?

ShawnnlearningIs it possible to find such a function satisfying: The function f is Riemann integrable on [0,1]; For ANY interval in [0,1], there are always both positive and negative values of $f(x)$. I didn't find any way to disprove such function but did't find a concrete example either.

0
Q: Any continuous function on a closed interval in $\mathbb{R}$ is integrable

luka5z Prove that any continuous function on a closed interval in $\mathbb{R}$ is integrable. Let $f:[a,b]\rightarrow \mathbb{R}$ be a continuous function. We want to show that for any $\epsilon>0$ there is $\delta>0$ such that whenever $S_1$ and $S_2$ are Riemann sums corresponding to partitions...

0
Q: $A \in M_3(\mathbb Z)$ be such that $\det(A)=1$ ; then what is the maximum possible number of entries of $A$ that are even ?

Saun DevLet $A \in M_3(\mathbb Z)$ be such that $\det(A)=1$ ; then what is the maximum possible number of entries of $A$ that are even ?

0
Q: Concave function

Eddie YuProve that three statements are equal. 1) f(x) is a concave function. 2) For all x, x*, f(x)= 3) For all x, D^2f(x)=H (Hessian) is negative semi-definite.

Short title. Short question. Concave function
0
Q: Dealing with "finding what some vector in the codomain is the image of"??

Jackie VazquezSo I'm having trouble finding a pattern when dealing with these types of questions; I need to find a better way to solve them: Here's the one i'm currently dealing with: Find the range space and rank of the map: a) f: R2 → P3 given by (x,y) --> (0, x-y, 3y) (these are vectors btw) So I ...

0
Q: Binomial distribution Question

Maths2468How do you solve this question: Consider two independent random variables X and Y such that X ~ B(2, a) and Y ~ B(2, b). Let W be the random variable that represents the product of each value of X with each value of Y.Construct a table showing the probability distribution of W. Hence find an exp...

Words such as question are uninformative in titles. Please edit the title so that it better describes the specifics of your question. Do not hesitate to make it longer or include a formula if needed. More tips here. (from a bot)Normal Human 48 secs ago
0
Q: Solution of an ODE

workaholicI am trying to solve the following ODE: $$y''(x)+\left( k_0-\frac{\lambda}{1+cosh^2(ax)}\right)y(x)=0 \, \, \,\,\, k_0,\lambda,a>0$$ when as $x \rightarrow \infty$ the solution is of the form $y(x)=e^{ik_0x}$. My attempt: I did the folllowing substitution $t=-cosh^2(ax)$ and the ODE which cam...

Short title. Solution of an ODE
0
Q: Lagrange interpolation formula

user374186332f(1) = -3f(-4) + 10f(-2) + 30f(2) + 5f(4) solve using Lagrange interpolation formula.i have tried to solve this question but still no solution found.if any one have solution regarding this question then please share it.

Short title. Question contains please. Lagrange interpolation formula
0
Q: Why can the first term of the Taylor series expansion of $cos(\theta)$ be written in the way below?

TakolWhy can the first term of the Taylor series expansion of $cos(\theta)$ be written as $cos(\theta_0 - (\theta - \theta_0) sin (\theta_0))$?

0
Q: $S$ be a collection of subsets of $\{1,...,100\}$ ; any two sets in $S$ has non-empty intersection , what is the maximum possible value of $|S|$?

Saun DevLet $S$ be a collection of subsets of $\{1,2,...,100\}$ such that any two sets in $S$ has non-empty intersection . Then what is the maximum possible cardinality of $S$ ?

0
Q: Mathematical tool for Image Comparision.

user2495765Dears, I had prepared a noisy image. I need to compare the original image w.r.t. noisy image. For this I found Pearson Correlation coefficient and SSIM (Structural Similarity Index). I wonder the difference between these two and which is best for Image comparison. Is there some other method to c...

Consider adding a tag for a broader subject area to which the question belongs. Some of these tags might fit. (from a bot)Normal Human 30 secs ago
0
Q: Prove that for all primes p: $φ(p^i)$= $p^i$ - $p^{i-1}$

welfeProve that for all primes p: $φ(p^i)$= $p^i$ - $p^{i-1}$ I found a proof on the wikipedia article of the Euler's totient function. But I cannot understand it, as it's been many years since I dealed with math and proves. Is there maybe a longer explanation somewhere, or can you explain it in deta...

 
3:24 PM
0
Q: In Given 10 letters, Five letter words are formed from these given letters at least 2 repetition

Ashish PatelTen different letters are given. Five letter words are formed from these given letters. The number of words having at least two letters repeated is ? Please explain your solution

0
Q: Does every smooth manifold admit a smooth $\Delta$-complex structure?

DavidI know that every smooth manifold admits a triangulation. Does this mean that it also admits a smooth $\Delta$-complex structure?

0
Q: equivalence relation of a set

user290335Hi can anyone help me on this problem? I have the set {0,1,3,8,9} and I want to define an example of an equivalence relation. I know that to be a equivalence relation it needs to be Reflexive, symmetric and transitive I also now that for a set of 5 elements there are 2^n^2, so if n=5 there ...

0
Q: Question about polynomial functions and conditions for a linear form to be a basis for V*

cathi_w92Let V be the vector space of all polynomial functions from R[x]<3 to R[x]<3 . Consider the linear forms f_i defined for p in V, as f_i(p)=p(a_i), where a_i is in R, for i in {1,2,3,4}. a) Determine under what conditions {f1,f2,f3,f4} is a basis for V*. b) Suppose that you have found the condition...

This site uses MathJax formatting of formulas. More tips here. (autocomment)Normal Human 33 secs ago
0
Q: Towers of Hanoi proof by induction

MichaelWe have a 3 labelled colours, blue, green, red. n of the labels are to be arranged in a line so that no two consecutive labels are both red. Let $H(n)$ be the number of ways this can be done. Explain why H(n) satisifies the following: $H(1)= 3, H(2) = 8, H(n)=2H(n-1) + 2H(n-2), \forall n \ge...

Tagged proof-explanation. Towers of Hanoi proof by induction
-1
Q: Is this a polynomial? Or shall I call it some other thing?

ZiloricLet V and W be finite dimensional vector spaces over R and let T_1 :V--->V and T_2:W--->W be two L.T's whose minimal polynomials are given by f(x)=x^3+x^2+x+1 and g(x)=x^4-x^2-2 let T :V+W--->V+W be LT defined by T(v,w)=(T_1(v) ,T_2(w)) for (v,w) in V+W and let h(x) be the polynomial of T. Wha...

This site uses MathJax formatting of formulas. More tips here. (from a bot)Normal Human 1 min ago
0
Q: quasi-concavity

Eddie Yuf:R n →R f:Rn→R prove that the two statements are equal 1) For all x,y x,y in R n Rn , and for all t in [0,1] [0,1] , f(tx+(1−t)y)>=minf(x),f(y) f(tx+(1−t)y)>=minf(x),f(y) 2) For all k k in R R , x:f(x)>=k x:f(x)>=k is a convex set.

Short title. quasi-concavity
0
Q: Prove that $\lim\limits_{n\to \infty} \sup t_n=\lim\limits_{n\to \infty} \sup t_n\sqrt[n]{n}$

Raheem NajibLet $t_n\ge 0$. How to prove that $\lim\limits_{n\to \infty} \sup t_n=\lim\limits_{n\to \infty} \sup t_n\sqrt[n]{n}$? Here is my sketch: So $t_n\ge 0$ and $\sqrt[n]{n}\ge 1$ then $t_n\sqrt[n]{n}\ge t_n$ hence $$\lim\limits_{n\to \infty} \sup t_n\sqrt[n]{n}\ge\lim\limits_{n\to \infty} \sup t_n.$$...

Title contains \limits, \limits. [Prove that $\lim\limits_{n\to \infty} \sup t_n=\lim\limits_{n\to \infty} \sup t_n\sqrt[n]{n}$](math.stackexchange.com/q/1573660)
 
3:48 PM
6
Q: Make the New Answers to Old Questions real time

Bhargav RaoThe 10k privileges has a great additional advantage of looking at New Answers to Old questions. However, the updates are not real time like the other pages on Stack Overflow. Additionally, there is no ability to flag an answer directly from the tool: you have to go to the post and flag it from t...

0
Q: Why the coma looks more a product when we use chemistry LaTeX?

ShadockI saw something strange here. When we use LaTeX in chemistry $0.85$ become $\ce{0.85}$ and then looks exactly the same as a product $a \cdot b$. Obviously here it's understandable but it bothered me when I saw it. Is it a bug? Why works it like this?

 
0
Q: Let U1 and U2 be independent random variables having the Uniform(0, 1) distribution

JohnFind the cumulative distribution function of X = U1/U2. I'm not really sure where to go from here. I've seen an example that uses the expected values and I'm not sure why.

0
Q: $A$ be a subset of $[0,1]$ with non-empty interior ; then is it true that $\mathbb Q+A=\mathbb R$?

Saun DevLet $A$ be a subset of $[0,1]$ with non-empty interior ; then is it true that $\mathbb Q+A=\mathbb R$ ?

Short question. [$A$ be a subset of $[0,1]$ with non-empty interior ; then is it true that $\mathbb Q+A=\mathbb R$?](math.stackexchange.com/q/1573666)
0
Q: Solving an equation with Maple results in "Warning, solutions may have been lost"

Desmond HumeI'm trying to solve an integral equation (on the screenshot) as a part of finding an answer to this question but Maple says "Warning, solutions may have been lost".

Consider adding a tag for a broader subject area to which the question belongs. Some of these tags might fit. (from a bot)Normal Human 58 secs ago
0
Q: evaluation of fourier transform of electric potential

PhysicistI would like to ask how to evaluate equation 7? I have spent hours and still have no idea how to get a(k).

-1
Q: locus of circles

SoroushWe have two circles (with optional radii) and one point outside of these 2 circles(optional position). how can we draw a circle or circles (locus' circles) so that pass the point and be tangent with two circles? (need a basic solution for finding all locuses)

Short title. locus of circles
0
Q: show that a serie converges

ShlomiProve that the serie: $$ \sum _{n=1} ^\infty \frac{(-1)^{\lfloor n/3 \rfloor}}{n} $$ I tryed to use Leibniz rule but I did not succeed. Any suggestions? Thanks for helpers!

Short title. Short question. show that a serie converges
0
Q: Why is this proof considered wrong?

HyunMiI was asked to prove following statement: $\log_a{(x_1.x_2)} = \log_a{(x_1)} + \log_a{(x_2)}$ What I did was: $\log_a{(x_1.x_2)} = \log_a{(x_1)} - (-1).\log_a{(x_2)}$ $\log_a{(x_1.x_2)} = \log_a{(x_1)} - \log_a{(\frac{1}{x_2})}$ $\log_a{(x_1.x_2)} = \log_a{(\frac{x_1}{\frac{1}{x_2}})}$ $\lo...

0
Q: Infinite trigonometric summation

PiyushCan the following summation be written in a finite number of terms or as an integral: $$/sum_{r=1}^{/infty}/frac{/tan(/theta/2^n)}{2^(r-1)/cos(/theta/2^(r-1))}$ I tried to simplify the expression using trigonometric identities and then converting the infinite summation into a definite integral...

0
Q: Prove an equation: Product of subsets

Marius KüpperI don't know how to prove that the following equation is correct: Equation to prove

0
Q: Why it is so difficult to perform primality test on huge fermat numbers?

BulldogisthebestdogI am not good in computer programming at all, but I know that it will take a lot of times to perform primality test on huge numbers (10 million or billion of digits). But I particularly get interested on fermat numbers,which is numbers of the form $2^{2^{n}}$$+$$1$. And the smallest fermat number...

Welcome to Math.SE, Bulldogisthebestdog. Consider adding a tag for a broader subject area to which the question belongs. Some of these tags might fit. (autocomment)Normal Human 42 secs ago
0
Q: I like math, but can't keep up with the pace.

TheoYouI'm a math major. I like math. I'm comfortable with it. I'm considering to do a PhD. The thing is I always fall behind. I can't keep up with the professor, can't turn in satisfactory homework in time, can't do well at tests either. When I learn something, I want to understand it. I want it...

Tag (soft-question) should not be the only tag a question has. Please add a tag for a subject area to which the question belongs. (from a bot)Normal Human 29 secs ago
0
Q: Membership Problem is undecidable?

Ali JafarIf Machine accepts the string 'W' then 'W' is the part of M other wise not. Prove that membership problem is undecidable

Title contains problem. Short question. Membership Problem is undecidable?
 
4:37 PM
0
Q: Two 'textbooks' tags

OokerI think we should burnimate the latter.

 
0
Q: prove or disprove: if E in R is a Null set, than its closer is also a null set.

tom schneorsonprove or disprove: if E in R is a Null set, than its closer is also a null set. I think its true. I have no sense of direction on how to start.. thanks in advance for any response..

0
Q: Solve the equation with trigonometry

prishila$$2cosx/(1+cos2x)=(1-cos2x)/sin2x$$ Here's what i did $$2cosx/(1+cos^2x-sin^2x)=(1-(cos^2x-sin^2x))/2sincosx$$ $$2cosx/2cos^x=(sin^2x+sin^2x)/2sinxcosx.$$ How do i continue?

0
Q: What is the smallest positive value of K which makes the closed-loop system unstable?

Roberto SousaYou are given a transfer function G(s)=1.81K(s+20)/(s3+10s2+32s+32). This system is connnected with unity negative feedback. I've tried so many things but I can't do it :( I've did 1.81K(s+20)=0, but it's clearly wrong. I've got the zeros for the bottom part (s=-2 or s=-4(2x)). So, 1.81K(s+20)/s...

Welcome to Math.SE, Roberto Sousa. This site uses MathJax formatting of formulas. More tips here. (from a bot)Normal Human 45 secs ago
0
Q: How do I know if my linear function(al)s are one-to-one, onto, both or neither?

CaydeI have come across a question: Define $f:M(F) \rightarrow F$, by $f(A)=tr(A)$. $M$ is n by n. For the first part, I have proven it was a linear functional. Done. Now, show $f$ is onto. Is $f$ one-to-one? What am I looking to do? I don't know what I'm even looking to prove. Walk me through it ...

0
Q: Explain factorial simplification, please

Jean-Yves LEMAIREProve that $\frac{n-b}{n}*\frac{n-b-1}{n-1}*\frac{n-b-2}{n-2}*...*\frac{n-b-(k-2)}{n-(k-2)}*\frac{b}{n-(k-1)}=\frac{(n-b)!}{(n-b-k+1)!}*\frac{(n-k)!*b}{n!}$

Welcome to Math.SE, Jean-Yves LEMAIRE. Words such as please are uninformative in titles. Please edit the title so that it better describes the specifics of your question. Do not hesitate to make it longer or include a formula if needed. More tips here. (from a bot)Normal Human 23 secs ago
-1
Q: find infimum and suprmum

Nir Movshovitz$x^{3} \geq 25$ and x is rational. I have find what is the inifimum and minimum of x. I know that there is no infimum because x is rational but i have to proove it somehow. I know there is some way to proove it with the rule that for every real number there is bigger natural number. Can you hel...

Welcome to Math.SE, Nir Movshovitz. Consider adding a tag for a broader subject area to which the question belongs. Some of these tags might fit. (from a bot)Normal Human 41 secs ago
0
Q: Eigenvalues and subspaces

TheNotMeSuppose that $v_1 \neq v_2 \neq ... \neq v_n$ are eigenvalues of a matrix $A$, $n>3$. We know that eigenvectors form a subspace of $R^n$. But is it true to say that, if we take a subset of these, for example $\{v_1,v_2,v_3\}$, those span a subspace of $R^n$ of dimension $3$?

0
Q: convert and solve integral into differential equation

Mohanad KaleiaI have the following integral: $F(t) = \int_0^tF(y)dy$ How can I convert that integral into differential equation and solve it?

0
Q: Helping understand manifolds

vanHohenheimi have a problem with understanding manifolds. Definition is quite unclear for me. Would be more than glad for intuitive explanation. And in addition i need to find whether $$M=\{(x,y,z): x^2y^2+y^2z^2+z^2x^2=xyz; x,y,z>0\}$$ is manifold or not. $F(x,y,z)=x^2y^2+y^2z^2+z^2x^2-xyz$ and $M=\{(x,y,z...

0
Q: When the integers got upset.

ishanI have been stuck with this problem for quite large time. https://www.hackerearth.com/code-monk-bit-manipulation/algorithm/when-the-integers-got-upset/. In short what is says is: There are two arrays A and P of length N. There is a third array Z whose values are calculated as follows: Z[i]=(A[i]...

 
00:00 - 17:0017:00 - 00:00

« first day (41 days earlier)      last day (533 days later) »